TransWikia.com

Entire function can be approximated uniformly by polynomials with rational coefficients on every compact set.

Mathematics Asked by JacobsonRadical on January 14, 2021

I am working on Stein Complex Analysis Chapter 2 Problem 5. The question has a hint that I cannot prove. Basically, the hint states as follows:

Let $p_{1}, p_{2},cdots$ denote an enumeration of the collection of polynomials whose coefficients have rational real and imaginary parts. Given $h$ entire, there exists a sequence ${n_{k}}$ such that $lim_{krightarrowinfty}p_{n_{k}}(z)=h(z)$ uniformly on every compact subset of $mathbb{C}$.

I understand that this is pretty similar to the Runge theorem:

Any function holomorphic in a neighborhood of a compact set $K$ can be approximated uniformly on $K$ by rational functions whose singularities are in $K^{c}=mathbb{C}setminus K$. If $K^{c}$ is connected, any function holomorphic in a neighborhood of $K$ can be approximated uniformly on $K$ by polynomials.

To prove the hint quickly, I tried to prove it using this theorem: as $h$ was entire, it is of course holomorphic in a neighborhood of any compact set $K$. Thus, it can be approximated by rational functions whose singularities are in $K^{c}$.

Then I got stuck. If $K^{c}$ is connected, then I can say $h(z)$ is approximated by a sequence of polynomial, and by density it is clear that this sequence of polynomial can be approximated by a sequence of polynomials with rational coefficients. Then we are done.

But generally speaking we do not have $K^{c}$ to be connected. What should I do?

One Answer

As per comments the crucial observation is that for every $n ge 1$ we can find a polynomial $P_{n,h}$ st $|P_{n,h}(z)-h(z)| < 1/(2n), |z| le n$ (eg some Taylor truncation of $h$).

But then since the closed disc is compact one can tweak the coefficients of $P_{n,h}$ very little and find $Q_{n,h}$ with rational coefficients st $|P_{n,h}(z)-Q_{n,h}(z)| < 1/(2n), |z| le n$

Putting it together we get the sequence $Q_{n,h}$ satisfying $|h(z)-Q_{n,h}(z)| < 1/n, |z| le n$. But now any compact $K$ is contained in $|z| le n$ for $n ge n_K$ so $Q_{n,h}$ converges uniformly on $K$ to $h$

Correct answer by Conrad on January 14, 2021

Add your own answers!

Ask a Question

Get help from others!

© 2024 TransWikia.com. All rights reserved. Sites we Love: PCI Database, UKBizDB, Menu Kuliner, Sharing RPP